Difference between revisions of "2021 JMPSC Invitationals Problems/Problem 9"

(Solution)
 
(4 intermediate revisions by the same user not shown)
Line 2: Line 2:
 
In <math>\triangle ABC</math>, let <math>D</math> be on <math>\overline{AB}</math> such that <math>AD=DC</math>. If <math>\angle ADC=2\angle ABC</math>, <math>AD=13</math>, and <math>BC=10</math>, find <math>AC.</math>
 
In <math>\triangle ABC</math>, let <math>D</math> be on <math>\overline{AB}</math> such that <math>AD=DC</math>. If <math>\angle ADC=2\angle ABC</math>, <math>AD=13</math>, and <math>BC=10</math>, find <math>AC.</math>
  
==Solution==
+
==Solution 1==
asdf
+
From the fact that <math>AD=DB</math> and <math>\angle ADC = 2\angle ABC,</math> we find that <math>\triangle ABC</math> is a right triangle with a right angle at <math>C;</math> thus by the Pythagorean Theorem we obtain <math>AC=\boxed{24}.</math> ~samrocksnature
 +
 
 +
==Solution 2 (Stewart's Theorem)==
 +
Note that <math>\angle BDC = 180-x</math>, which means <math>\angle DCB = \angle DBC</math> and <math>AD=DB=DC=13</math>. Now, Stewart's Theorem dictates <math>x^2 \cdot 13 = 7488</math>, yielding <math>AC=x=\boxed{24}</math> ~Geometry285
  
 
==See also==
 
==See also==
#[[2021 JMPSC Invitational Problems|Other 2021 JMPSC Invitational Problems]]
+
#[[2021 JMPSC Invitationals Problems|Other 2021 JMPSC Invitationals Problems]]
#[[2021 JMPSC Invitational Answer Key|2021 JMPSC Invitational Answer Key]]
+
#[[2021 JMPSC Invitationals Answer Key|2021 JMPSC Invitationals Answer Key]]
 
#[[JMPSC Problems and Solutions|All JMPSC Problems and Solutions]]
 
#[[JMPSC Problems and Solutions|All JMPSC Problems and Solutions]]
 
{{JMPSC Notice}}
 
{{JMPSC Notice}}

Latest revision as of 18:10, 11 July 2021

Problem

In $\triangle ABC$, let $D$ be on $\overline{AB}$ such that $AD=DC$. If $\angle ADC=2\angle ABC$, $AD=13$, and $BC=10$, find $AC.$

Solution 1

From the fact that $AD=DB$ and $\angle ADC = 2\angle ABC,$ we find that $\triangle ABC$ is a right triangle with a right angle at $C;$ thus by the Pythagorean Theorem we obtain $AC=\boxed{24}.$ ~samrocksnature

Solution 2 (Stewart's Theorem)

Note that $\angle BDC = 180-x$, which means $\angle DCB = \angle DBC$ and $AD=DB=DC=13$. Now, Stewart's Theorem dictates $x^2 \cdot 13 = 7488$, yielding $AC=x=\boxed{24}$ ~Geometry285

See also

  1. Other 2021 JMPSC Invitationals Problems
  2. 2021 JMPSC Invitationals Answer Key
  3. All JMPSC Problems and Solutions

The problems on this page are copyrighted by the Junior Mathematicians' Problem Solving Competition. JMPSC.png